K
Khách

Hãy nhập câu hỏi của bạn vào đây, nếu là tài khoản VIP, bạn sẽ được ưu tiên trả lời.

31 tháng 5 2020

Ta có:

\(\frac{1}{a^2+1}+\frac{1}{b^2+1}+\frac{1}{c^2+1}+\frac{1}{d^2+1}\)

= \(\left(1-\frac{a^2}{a^2+1}\right)+\left(1-\frac{b^2}{b^2+1}\right)+\left(1-\frac{c^2}{c^2+1}\right)+\left(1-\frac{d^2}{d^2+1}\right)\)

= \(4-\left(\frac{a^2}{a^2+1}+\frac{b^2}{b^2+1}+\frac{c^2}{c^2+1}+\frac{d^2}{d^2+1}\right)\)

Áp dụng Cô - si:

\(a^2+1\ge2\sqrt{a^2.1}=2a\) <=> \(\frac{a^2}{a^2+1}\le\frac{a}{2}\)

Tương tự => \(\left\{{}\begin{matrix}\frac{b^2}{b^2+1}\le\frac{b}{2}\\\frac{c^2}{c^2+1}\le\frac{c}{2}\\\frac{d^2}{d^2+1}\le\frac{d}{2}\end{matrix}\right.\)

<=> \(4-\left(\frac{a^2}{a^2+1}+\frac{b^2}{b^2+1}+\frac{c^2}{c^2+1}+\frac{d^2}{d^2+1}\right)\)

\(\ge4-\frac{a+b+c+d}{2}=2\)

1 tháng 8 2020

Xét \(\frac{a^3}{a^2+ab+b^2}-\frac{b^3}{a^2+ab+b^2}=\frac{\left(a-b\right)\left(a^2+ab+b^2\right)}{a^2+ab+b^2}=a-b\)

Tương tự, ta được: \(\frac{b^3}{b^2+bc+c^2}-\frac{c^3}{b^2+bc+c^2}=b-c\)\(\frac{c^3}{c^2+ca+a^2}-\frac{a^3}{c^2+ca+a^2}=c-a\)

Cộng theo vế của 3 đẳng thức trên, ta được: \(\left(\frac{a^3}{a^2+ab+b^2}+\frac{b^3}{b^2+bc+c^2}+\frac{c^3}{c^2+ca+a^2}\right)\)\(-\left(\frac{b^3}{a^2+ab+b^2}+\frac{c^3}{b^2+bc+c^2}+\frac{a^3}{c^2+ca+a^2}\right)=0\)

\(\Rightarrow\frac{a^3}{a^2+ab+b^2}+\frac{b^3}{b^2+bc+c^2}+\frac{c^3}{c^2+ca+a^2}\)\(=\frac{b^3}{a^2+ab+b^2}+\frac{c^3}{b^2+bc+c^2}+\frac{a^3}{c^2+ca+a^2}\)

Ta đi chứng minh BĐT phụ sau: \(a^2-ab+b^2\ge\frac{1}{3}\left(a^2+ab+b^2\right)\)(*)

Thật vậy: (*)\(\Leftrightarrow\frac{2}{3}\left(a-b\right)^2\ge0\)*đúng*

\(\Rightarrow2LHS=\Sigma_{cyc}\frac{a^3+b^3}{a^2+ab+b^2}=\Sigma_{cyc}\text{ }\frac{\left(a+b\right)\left(a^2-ab+b^2\right)}{a^2+ab+b^2}\)\(\ge\Sigma_{cyc}\text{ }\frac{\frac{1}{3}\left(a+b\right)\left(a^2+ab+b^2\right)}{a^2+ab+b^2}=\frac{1}{3}\text{​​}\Sigma_{cyc}\left[\left(a+b\right)\right]=\frac{2\left(a+b+c\right)}{3}\)

\(\Rightarrow LHS\ge\frac{a+b+c}{3}=RHS\)(Q.E.D)

Đẳng thức xảy ra khi a = b = c

P/S: Có thể dùng BĐT phụ ở câu 3a để chứng minhxD:

27 tháng 7 2020

1) ta chứng minh được \(\Sigma\frac{a^4}{\left(a+b\right)\left(a^2+b^2\right)}=\Sigma\frac{b^4}{\left(a+b\right)\left(a^2+b^2\right)}\)

\(VT=\frac{1}{2}\Sigma\frac{a^4+b^4}{\left(a+b\right)\left(a^2+b^2\right)}\ge\frac{1}{4}\Sigma\frac{a^2+b^2}{a+b}\ge\frac{1}{8}\Sigma\left(a+b\right)=\frac{a+b+c+d}{4}\)

bài 2 xem có ghi nhầm ko

Câu 1: Cho \(a,b,c0\)và \(a+b+c=3\). Chứng minh rằng:\(\frac{a}{1+b^2}+\frac{b}{1+c^2}+\frac{c}{1+a^2}\ge\frac{3}{2}\).Câu 2: Cho \(a,b,c,d0\)và \(a+b+c+d=4\). Chứng minh rằng:\(\frac{a}{1+b^2}+\frac{b}{1+c^2}+\frac{c}{1+d^2}+\frac{d}{1+a^2}\ge2\).Câu 3: Cho \(a,b,c,d0\). Chứng minh rằng:\(\frac{a^3}{a^2+b^2}+\frac{b^3}{b^2+c^2}+\frac{c^3}{c^2+d^2}+\frac{d^3}{d^2+a^2}\ge\frac{a+b+c+d}{2}\).Câu 4: Cho \(a,b,c,d0\). Chứng minh...
Đọc tiếp

Câu 1Cho \(a,b,c>0\)và \(a+b+c=3\). Chứng minh rằng:

\(\frac{a}{1+b^2}+\frac{b}{1+c^2}+\frac{c}{1+a^2}\ge\frac{3}{2}\).

Câu 2: Cho \(a,b,c,d>0\)và \(a+b+c+d=4\). Chứng minh rằng:

\(\frac{a}{1+b^2}+\frac{b}{1+c^2}+\frac{c}{1+d^2}+\frac{d}{1+a^2}\ge2\).

Câu 3: Cho \(a,b,c,d>0\). Chứng minh rằng:

\(\frac{a^3}{a^2+b^2}+\frac{b^3}{b^2+c^2}+\frac{c^3}{c^2+d^2}+\frac{d^3}{d^2+a^2}\ge\frac{a+b+c+d}{2}\).

Câu 4: Cho \(a,b,c,d>0\). Chứng minh rằng:

\(\frac{a^4}{a^3+2b^3}+\frac{b^4}{b^3+2c^3}+\frac{c^4}{c^3+2d^3}+\frac{d^4}{d^3+2a^3}\ge\frac{a+b+c+d}{3}\).

Câu 5: Cho \(a,b,c>0\)và \(a+b+c=3\). Chứng minh rằng:

\(\frac{a^2}{a+2b^2}+\frac{b^2}{b+2c^2}+\frac{c^2}{c+2a^2}\ge1\).

Câu 6: Cho \(a,b,c>0\)và \(a+b+c=3\). Chứng minh rằng: 

\(\frac{a^2}{a+2b^3}+\frac{b^2}{b+2c^3}+\frac{c^2}{c+2a^3}\ge1\).

Câu 7: Cho \(a,b,c>0\)và \(a+b+c=3\). Chứng minh rằng:

\(\frac{a+1}{b^2+1}+\frac{b+1}{c^2+1}+\frac{c+1}{a^2+1}\ge3\).

Câu 8: Cho \(a_1,a_2,...,a_{n-1},a_n>0\)và \(a_1+a_2+...+a_{n-1}+a_n=n\)với \(n\)nguyên dương. Chứng minh:

\(\frac{1}{a_1+1}+\frac{1}{a_2+1}+...+\frac{1}{a_{n-1}+1}+\frac{1}{a_n+1}\ge\frac{n}{2}\).

 

 

0
NV
16 tháng 11 2019

a/ Đề sai, đề đúng phải là \(p=\frac{a+b+c}{2}\)

b/ \(\Leftrightarrow\frac{2}{2+a^2b}+\frac{2}{2+b^2c}+\frac{2}{2+c^2a}\ge2\)

\(VT=1-\frac{a^2b}{1+1+a^2b}+1-\frac{b^2c}{1+1+b^2c}+1-\frac{c^2a}{1+1+c^2a}\)

\(VT\ge3-\left(\frac{a^2b}{3\sqrt[3]{a^2b}}+\frac{b^2c}{3\sqrt[3]{b^2c}}+\frac{c^2a}{3\sqrt[3]{c^2a}}\right)\)

\(VT\ge3-\frac{1}{9}\left(3\sqrt[3]{a^2.ab.ab}+3\sqrt[3]{b^2.bc.bc}+3\sqrt[3]{c^2.ca.ca}\right)\)

\(VT\ge3-\frac{1}{9}\left(a^2+2ab+b^2+2bc+c^2+2ca\right)\)

\(VT\ge3-\frac{1}{9}\left(a+b+c\right)^2=2\) (đpcm)

Dấu "=" xảy ra khi \(a=b=c=1\)

6 tháng 9 2016

\(N=\frac{a}{1+b^2c}+\frac{b}{1+c^2d}+\frac{c}{1+d^2a}+\frac{d}{1+a^2b}\)

Áp dụng BĐT Cauchy ta có:

\(\frac{a}{1+b^2c}=a-\frac{ab^2c}{1+b^2c}\)

\(\ge a-\frac{ab^2c}{2b\sqrt{c}}=a-\frac{ab\sqrt{c}}{2}=a-\frac{b\sqrt{ac}\sqrt{a}}{2}\)

\(\ge a-\frac{b\left(ac+c\right)}{4}\).Suy ra \(\frac{a}{1+b^2c}\ge a-\frac{1}{4}\cdot\left(ab+abc\right)\)

Tương tự ta có:

\(\frac{b}{a+c^2d}\ge b-\frac{1}{4}\left(bc+bcd\right)\)

\(\frac{c}{1+d^2a}\ge c-\frac{1}{4}\left(cd+cda\right)\)

\(\frac{d}{1+a^2b}\ge d-\frac{1}{4}\left(da+dab\right)\)

Do đó: \(S=\frac{a}{1+b^2c}+\frac{b}{1+c^2d}+\frac{c}{1+d^2a}+\frac{d}{1+a^2b}\)

\(\ge a+b+c+d-\frac{1}{4}\left(ab+bc+cd+da+abc+bcd+cda+dab\right)\)

\(=4-\frac{1}{4}\left(ab+bc+cd+da+abc+bcd+cda+dab\right)\)

Ta có:

\(ab+bc+cd+da\le\frac{1}{4}\left(a+b+c+d\right)^2=4\)

\(abc+bcd+cda+dab\le\frac{1}{16}\left(a+b+c+d\right)^3=4\)

nên \(S\ge4-\frac{1}{4}\cdot\left(4+4\right)=2\)(Đpcm)

Dấu = khi \(a=b=c=d=1\)

 

 

 

7 tháng 9 2016

tick đê =))

7 tháng 8 2019

Áp dụng BĐT cosi ta có

\(\frac{1}{a^3}+\frac{1}{a^3}+\frac{1}{b^3}\ge\frac{3}{a^2b}\)\(\frac{1}{b^3}+\frac{1}{b^3}+\frac{1}{c^3}\ge\frac{3}{b^2c}\)\(\frac{1}{c^3}+\frac{1}{c^3}+\frac{1}{d^3}\ge\frac{3}{c^2d}\)

\(\frac{1}{d^3}+\frac{1}{d^3}+\frac{1}{a^3}\ge\frac{3}{d^2a}\)

Cộng các BĐt trên ta có 

\(\frac{1}{a^3}+\frac{1}{b^3}+\frac{1}{c^3}+\frac{1}{d^3}\ge\frac{1}{a^2b}+\frac{1}{b^2c}+\frac{1}{c^2d}+\frac{1}{d^2a}\)(1)

Áp dụng BĐT buniacoxki ta có

\(\left(\frac{a^2}{b^5}+\frac{b^2}{c^5}+\frac{c^2}{d^5}+\frac{d^2}{a^5}\right)\left(\frac{1}{a^2b}+\frac{1}{b^2c}+\frac{1}{c^2d}+\frac{1}{d^2a}\right)\ge \left(\frac{1}{a^3}+\frac{1}{b^3}+\frac{1}{c^3}+\frac{1}{d^3}\right)^2\)

Kết hợp với (1)  ta được ĐPCM

Dấu bằng xảy ra khi a=b=c